The distance it takes a truck to stop can be

modeled by the function

Determine the value off, rounde

hundredth.

2

DONE

2.150 ?

d(u)=-

64.4f

d = stopping distance in feet

y = initial velocity in miles per hour

f= a constant related to friction

When the truck's initial velocity on dry pavement is

40 mph, its stopping distance is 138 ft.

Answers

Answer 1

Your question is not well presented (Refer below for correct question)

The distance it takes a truck to stop can be modeled by the function;

d(u) = 2.15u²/64.4f

Where

d = stopping distance in feet

u = initial velocity in miles per hour

f = a constant related to friction

When the truck's initial velocity on dry pavement is 40 mph, its stopping distance is 138 ft.

Determine the value of f.

Answer:

The friction constant is approximately 0.3871

Step-by-step explanation:

Given

Stopping distance, d(u) = 138ft

Initial Velocity, u = 40mph

Required

Friction constant, f.

To get the value of f, we need to simply substitute 40 for u and 138 for d(u) in the above expression.

In other words;

d(u) = 2.15u²/64.4f becomes

138 = 2.15 * 40²/64.4f

138 = 2.15 * 1600/64.4f

138 = 3440/64.4f

Multiply both sides by 64.4f

138 * 64.4f = 64.4f * 3440/64.4f

8887.2f = 3440

Divide both sides by 8887.2

8887.2f/8887.2 = 3440/8887.2

f = 3440/8887.2

f = 0.387073544

f = 0.3871 (Approximated)

Hence, the friction constant is approximately 0.3871

Answer 2

Answer:

first one is  .39
second answer is C   the last  one

Step-by-step explanation:

The Distance It Takes A Truck To Stop Can Bemodeled By The FunctionDetermine The Value Off, Roundehundredth.2DONE2.150

Related Questions

The city of Ventura would like to build a seawall to protect the city from the threat of tsunamis. Each additional inches of height further protects the city and the 100 residents are each willing to pay $10 per inch of seawall height, regardless of how many inches are provided. The cost of building a wall that is i inches high is given by c(i) = 6i^2. What is the Pareto Optimal height for the seawall?

Answers

Answer:

The Pareto Optimal height is  [tex]i = 100 \ inch[/tex]

Step-by-step explanation:

The Pareto Optimal height is a height  of the seawall at which an increase in wall height will exceed the amount the resident are willing to pay and a decrease will affect the protection of the city

The number of residents is [tex]n = 100[/tex]

The amount each are willing to pay is  [tex]z=[/tex]$10 per inch

 The cost of building a wall that is i inches high is given by [tex]c(i) = 6i^2.[/tex]

The total amount the residents are willing to pay is

          [tex]n = 100 * 10[/tex] =  $1000

The maximum cost  is mathematically represented as

                [tex]\frac{dc(i)}{di} = 10i[/tex]

which implies that

          1000 =  10i

Hence the Pareto Optimal height is

=>         [tex]i = \frac{1000}{10}[/tex]

             [tex]i = 100 \ inch[/tex]

Two dice are rolled. E is the event that the sum is even, F is the event of rolling at least one six, and G is the event that the sum is eight. List the outcomes for the following events:

a. E ∩ F {(2, 2), (4, 4), (6, 6)} {(6, 2), (6, 4), (6, 6), (2, 6), (4, 6)} {(2, 6), (4, 6), (6, 6)} ∅
b. Ec ∩ G {(6, 2), (6, 4), (6, 6), (2, 6), (4, 6)} {(2, 6), (4, 6), (6, 6)} {(2, 2), (4, 4), (6, 6)} ∅

Answers

Answer:

(a)[tex]E \cap F ={(2, 6),(4, 6),(6, 2),(6, 4),(6, 6)}[/tex]

(b) [tex]E^c \cap G =\{ \}[/tex]

Step-by-step explanation:

The sample space of two dice rolled is given below:

[tex]\{(1, 1), (1, 2), (1, 3), (1, 4), (1, 5), (1, 6)\\(2, 1), (2, 2), (2, 3), (2, 4), (2, 5), (2, 6)\\(3, 1), (3, 2), (3, 3), (3, 4), (3, 5), (3, 6)\\(4, 1), (4, 2), (4, 3), (4, 4), (4, 5), (4, 6)\\(5, 1), (5, 2), (5, 3), (5, 4), (5, 5),(5,6)\\(6, 1), (6, 2), (6, 3), (6, 4), (6,5), (6,6)\}[/tex]

For Event E (The sum is even), the outcomes are:

[tex](1, 1), (1, 3), (1, 5),(2, 2), (2, 4), (2, 6)\\(3, 1), (3, 3), (3, 5), (4, 2), (4, 4), (4, 6)\\(5, 1),(5, 3), (5, 5), (6, 2), (6, 4), (6,6)[/tex]

For Event F (Rolling at least one six), the outcomes are:

[tex](1, 6), (2, 6), (3, 6), (4, 6),(5,6),(6, 1), (6, 2), (6, 3), (6, 4), (6,5), (6,6)[/tex]

For Event G (The sum is eight), the outcomes are:

[tex](2, 6), (3, 5),(4, 4), (5, 3),(6, 2)[/tex]

(a)[tex]E \cap F[/tex]

Therefore:

[tex]E \cap F ={(2, 6),(4, 6),(6, 2),(6, 4),(6, 6)}[/tex]

(b)[tex]E^c \cap G[/tex]

E is the event that the sum is even

Therefore: [tex]E^c$ is the event that the sum is odd.[/tex]

Since G is the event that the sum is eight( which is even), the intersection of the complement of E and G will be empty.

Therefore:

[tex]E^c \cap G =\{ \}[/tex]

Harriet spins this 6 colour spinner and flips a coin.

What is the probability of getting grey and tails?

Answers

Answer: 9%

Step-by-step explanation:

In probabilities; when two or more conditions must be met, the "and" operator is used. The probability of grey and tails is 1/12

From the attached spinner, we have:

[tex]Grey = 1[/tex]

[tex]n =6[/tex] --- partitions

So, the probability of landing on grey is:

[tex]P(Grey) = \frac{Grey}{n}[/tex]

This gives:

[tex]P(Grey) = \frac{1}{6}[/tex]

In a coin, we have:

[tex]Tail = 1[/tex]

[tex]n =2[/tex] --- faces

So, the probability of tail is:

[tex]P(Tail) = \frac{Tail}{n}[/tex]

[tex]P(Tail) = \frac{1}{2}[/tex]

The probability of grey and tail is:

[tex]Pr = P(Grey) \times P(Tail)[/tex]

[tex]Pr = \frac{1}{6} \times \frac{1}{2}[/tex]

[tex]Pr = \frac{1}{12}[/tex]

Hence, the probability of getting grey and tail is 1/12

Read more on probability at:

https://brainly.com/question/13957582

What’s the correct answer for this?

Answers

5x + 34 = -2(1 - 7x)
5x + 34 = -2 + 14x
36 = 9x
x = 4
NO = -2(1 - 7*4)
NO = -2(1 - 28)
NO = -2 + 56
NO = 54

Answer:

54

Step-by-step explanation:

Since diameter AB divides MN into two equal parts hence

MO = NO

NOW,

5x+34 = -2(1-7x)

5x+34 = -2+14x

34+2 = 14x-5x

36 = 9x

Dividing both sides by 9

x = 4

Now,

NO = -2(1-7(4))

NO = -2+56

NO = -2+56

NO = 54

A motor oil retailer needs to fill 40 one quart bottles, and he has two tanks: one that contains 12 gallons of oil and one that contains 2 gallons of oil. Which will he need to fill the bottles?

Answers

Answer:12 gallons

Step-by-step explanation: It is enough to fill 40

The number of gallons in 40 quarts will be 10 gallons. Then the one that contains 12 gallons of oil.

What is Algebra?

Algebra is the study of abstract symbols, while logic is the manipulation of all those ideas.

Unit modification is the process of converting the measurement of a given amount between various units, often by multiplicative constants that alter the value of the calculated quantity without altering its impacts.

A motor oil retailer needs to fill 40 one-quart bottles.

We know that in 1 gallon, there are 4 quarts. Then the number of gallons in 40 quarts will be given as,

⇒ 40 / 4

⇒ 10 gallons

The number of gallons in 40 quarts will be 10 gallons. Then the one that contains 12 gallons of oil.

More about the Algebra link is given below.

https://brainly.com/question/953809

#SPJ2

A baseball diamond is a square with side 90 ft. A batter hits the ball and runs toward first base with a speed of 31 ft/s. (a) At what rate is his distance from second base decreasing when he is halfway to first base? (Round your answer to one decimal place.) ft/s (b) At what rate is his distance from third base increasing at the same moment? (Round your answer to one decimal place.)

Answers

Answer:

Step-by-step explanation:

Given that :

the side of the square = 90ft

The speed of the runner = 31 ft/sec

By the time the runner is halfway to the first base;  the distance covered by the runner in time(t)  is (31 t) ft and the distance half the base = 90/2 = 45 ft

Thus; 31 t = 45

t = 45/31

From the second base ; the distance is given as:

P² = (90)² + (90 - 31t )²  

P = [tex]\sqrt{(90)^2 + (90 - 31t )^2}[/tex]

By differentiation with time;

[tex]\dfrac{dP}{dt} =\dfrac{1}{ 2 \sqrt{90^2 +(90-31t)^2} } *(0+ 2 (90-31t)(0-31))[/tex]

[tex]\dfrac{dP}{dt} =\dfrac{1}{ 2 \sqrt{90^2 +(90-31t)^2} } * 2 (-31)(90-31t)[/tex]

At t = 45/31

[tex]\dfrac{dP}{dt} =\dfrac{1}{ 2 \sqrt{90^2 +45^2} } * 2 (-31)(45)[/tex]

[tex]\dfrac{dP}{dt} =\dfrac{-35*45}{100.623}[/tex]

= - 13.86 ft/sec

Hence, we can conclude that  as soon as the runner  is halfway to the first base, the distance to the second base is therefore decreasing by 13.86 ft/sec

b) The distance from third base can be expressed by the relation:

q² = (31t)² + (90)²

[tex]q = \sqrt{(31t)^2+(90)^2}[/tex]

By differentiation with respect to time:

[tex]\dfrac{dq}{dt} = \dfrac{1}{2\sqrt{90^2 + (31)t^2} } *(0+31^2 + 2t)[/tex]

At t = 45/31

[tex]\dfrac{dq}{dt} = \dfrac{1}{2\sqrt{90^2 + 45^2} } *(0+31^2 + \frac{45}{31})[/tex]

[tex]= \dfrac{31*45}{100.623}[/tex]

[tex]= 13.86 \ ft/sec[/tex]

Thus, the rate at which the runner's distance is from the third base is increasing at the same moment of 13.86 ft/sec. So therefore; he is moving away from the third base at the same speed to the first base)

a) The distance from second base is decreasing when the batter is halfway to first base at a rate of 13.9 feet per second.

b) The distance from third base is increasing when the batter is halfway to first base at a rate of 13.9 feet per second.

a) As the batter runs towards the first base, both the distance from second base and the length of the line segment PQ decrease in time. The distance from the second base is determined by Pythagorean theorem:

[tex]QS^{2} = QP^{2}+PS^{2}[/tex] (1)

By differential calculus we derive an expression for the rate of change of the distance from second base ([tex]\dot QS[/tex]), in feet per second:

[tex]2\cdot QS \cdot \dot{QS} = 2\cdot QP\cdot \dot{QP} + 2\cdot PS\cdot \dot {PS}[/tex]

[tex]\dot{QS} = \frac{QP\cdot \dot QP + PS\cdot \dot{PS}}{QS}[/tex]

[tex]\dot {QS} = \frac{QP\cdot \dot {QP}+PS\cdot \dot {PS}}{\sqrt{QP^{2}+PS^{2}}}[/tex] (2)

If we know that [tex]QP = 0.5L[/tex], [tex]PS = L[/tex], [tex]L = 90\,ft[/tex], [tex]\dot {QP} = -31\,\frac{ft}{s}[/tex] and [tex]\dot {PS} = 0\,\frac{ft}{s}[/tex], then the rate of change of the distance from second base is:

[tex]\dot {QS} = \frac{(45\,ft)\cdot \left(-31\,\frac{ft}{s} \right)}{\sqrt{(45\,ft)^{2}+(90\,ft)^{2}}}[/tex]

[tex]\dot {QS} \approx -13.864\,\frac{ft}{s}[/tex]

The distance from second base is decreasing when the batter is halfway to first base at a rate of 13.9 feet per second.

b) As the batter runs towards the first base, both the distance from third base increases and the distance from home increase in time. The distance from the third base is determined by Pythagorean theorem:

[tex]QT^{2} = HT^{2}+QH^{2}[/tex] (3)

By differential calculus we derive an expression for the rate of change of the distance from third base ([tex]\dot QT[/tex]), in feet per second:

[tex]2\cdot QT\cdot \dot{QT} = 2\cdot HT\cdot \dot {HT} + 2\cdot QH\cdot \dot {QH}[/tex]

[tex]\dot {QT} = \frac{HT\cdot \dot {HT}+QH\cdot \dot {QH}}{QT}[/tex]

[tex]\dot {QT} = \frac{HT\cdot \dot {HT}+QH\cdot \dot {QH}}{\sqrt{HT^{2}+QH^{2}}}[/tex]

If we know that [tex]HT = 90\,ft[/tex], [tex]QH = 45\,ft[/tex], [tex]L = 90\,ft[/tex], [tex]\dot{HT} = 0\,\frac{ft}{s}[/tex] and [tex]\dot {QH} = 31\,\frac{ft}{s}[/tex], then the rate of change of the distance from third base is:

[tex]\dot{QT} = \frac{(45\,ft)\cdot \left(31\,\frac{ft}{s} \right)}{\sqrt{(90\,ft)^{2}+(45\,ft)^{2}}}[/tex]

[tex]\dot{QT} \approx 13.864\,\frac{ft}{s}[/tex]

The distance from third base is increasing when the batter is halfway to first base at a rate of 13.9 feet per second.

We kindly invite to check this question on rates of change: https://brainly.com/question/13103052

A group of hikers buy 8 bags of trail mix. Each bag contains 3 1/2 cups of trail mix. The trail mix is shared evenly among 12 hikers. How many cups of trail mix will each hiker receive?

Each hiker will receive cups of trail mix.

Answers

Answer:

Each hiker will get 2 1/3 cups

Step-by-step explanation:

8 bags multiplied by 3.5

28 cups overall

The 28 cups is shared between 12 hikers

28/12=2 1/3 cups

India is the second most populous country in the world, with a population in 2008 of about 1.14 billion people. The population is growing by about 1.34% each year. If the population continues following this trend, during what year will the population reach 2 billion?

Answers

Answer:

India's population will reach 2 billion during the year of 2050.

Step-by-step explanation:

India's population in t years after 2008 is modeled by the following equation:

[tex]P(t) = P(0)(1+r)^{t}[/tex]

In which P(0) is the population in 2008 and r is the growth rate, as a decimal.

Population in 2008 of about 1.14 billion people. The population is growing by about 1.34% each year.

This means that [tex]P(0) = 1.14, r = 0.0134[/tex]. So

[tex]P(t) = P(0)(1+r)^{t}[/tex]

[tex]P(t) = 1.14(1+0.0134)^{t}[/tex]

[tex]P(t) = 1.14(1.0134)^{t}[/tex]

If the population continues following this trend, during what year will the population reach 2 billion?

t years after 2008.

t is found when P(t) = 2. So

[tex]P(t) = 1.14(1.0134)^{t}[/tex]

[tex]2 = 1.14(1.0134)^{t}[/tex]

[tex](1.0134)^{t} = \frac{2}{1.14}[/tex]

[tex]\log{(1.0134)^{t}} = \log{\frac{2}{1.14}}[/tex]

[tex]t\log{1.0134} = \log{\frac{2}{1.14}}[/tex]

[tex]t = \frac{\log{\frac{2}{1.14}}}{\log{1.0134}}[/tex]

[tex]t = 42.23[/tex]

2008 + 42 = 2050

India's population will reach 2 billion during the year of 2050.

Some people say Mauna Kea is a taller mountain than Mt. Everest, because its total under sea and above sea level height is

a0 meters.

Answers

Answer:

Mauna Kea has from the base that is to say the deepest 10 km of height that is to say 10,000 m which is equivalent to being higher than Mount Everest.jj

Step-by-step explanation:

 The main reason why we could say that Mauna Kea is the highest mountain, even surpassing Mount Everest, is because of its altitude if we compare them we see the difference; for example Mauna Kea from its base that is to say from the deepest the ocean measures 10,000 m of altitude, on the other hand the Mount Everest has an altitude of 8848 meters (29,029 ft) above sea level, that is to say from its base it measures 8848 m altitude.

Alexander threw a dart at this board a total of 40 times. Predict the number of times the dart will land on the number 3.
PLZ HURRY

Answers

Answer:

5% chance so 2 times

Step-by-step explanation:

Got chu:)

Darby states this generalization:
All odd numbers greater than 30 are divisible by 3.
Which number could be used to show that Darby's generalization is not correct?
A. 33
B .35
C. 39
D. 45​

PLEASE HELP AS QUICKLY AS POSSIBLE THANK YOU :)

Answers

Answer:

35

Step-by-step explanation:

35 is not divisible by 3 as all the other numbers are

Answer: The number to show he is incorrect is 35 because 35 divided by 3 is 11.6666666667 which is not a whole number.

Step-by-step explanation:

2. Suppose you obtain a $3,000 T - note with a 3% annual rate, paid quarterly, with maturity in 5 years. How much interest will you earn?

Answers

Answer:

You will earn $483.55 in interest.

Step-by-step explanation:

The compound interest formula is given by:

[tex]A(t) = P(1 + \frac{r}{n})^{nt}[/tex]

Where A(t) is the amount of money after t years, P is the principal(the initial sum of money), r is the interest rate(as a decimal value), n is the number of times that interest is compounded per year and t is the time in years for which the money is invested or borrowed.

$3,000 T - note with a 3% annual rate

This means that [tex]P = 3000, r = 0.03[/tex]

Paid quarterly

Quarterly is 4 times per year, so [tex]n = 4[/tex]

Maturity in 5 years.

This means that [tex]t = 5[/tex]

How much interest will you earn?

Interest is the final amount subtracted by the principal.

Final amount:

A(5).

[tex]A(t) = P(1 + \frac{r}{n})^{nt}[/tex]

[tex]A(5) = 3000(1 + \frac{0.03}{4})^{4*5}[/tex]

[tex]A(5) = 3483.55[/tex]

Interest:

$3,483.55 - $3,000 = $483.55

You will earn $483.55 in interest.

Find the roots of the polynomial function f(x)=x^3+2x^2+x

Answers

Answer:

0 and  -1

Step-by-step explanation:

Hello,

[tex]x^3+2x^2+x = x(x^2+2x+1)=x(x+1)^2[/tex]

so the roots are

0 and -1 (to get x+1=0 )

do not hesitate if you need further explanation

hope this helps

Answer:

It's actually X = 0, X = - 1, with multiplicity 2, Option D, on Edge2020

Step-by-step explanation:

I just did it on edge :)

Which statements are true about reflections? Check all that apply.

An image created by a reflextion will always be congruent to its pre-image.

An image and its pre-image are always the same distance from the line of reflection.

If a point on the pre-image lies on the line of reflection, the image of that point is the same as the pre-image.

The line of reflection is perpendicular to the line segments connecting corresponding vertices.

The line segments connecting corresponding vertices are all congruent to each other.

The line segments connecting corresponding vertices are all parallel to each other.

Answers

1,2,3, and 5 are correct

Each contestant in the Hunger Games must be trained to compete. Suppose that the time it takes to train a contestant has mean 5 days and standard deviation 4 days, independent of the time it takes other contestants to train. If the Hunger Games has 100 contestants to train, approx imate the probability that it will take less than 450 days to train all the contestants. Leave your answer in terms of the standard normal distribution phi(a).

Answers

Answer:

11.51% probability that it will take less than 450 days to train all the contestants.

Step-by-step explanation:

To solve this question, we need to understand the normal probability distribution and the central limit theorem.

Normal probability distribution

When the distribution is normal, we use the z-score formula.

In a set with mean [tex]\mu[/tex] and standard deviation [tex]\sigma[/tex], the zscore of a measure X is given by:

[tex]Z = \frac{X - \mu}{\sigma}[/tex]

The Z-score measures how many standard deviations the measure is from the mean. After finding the Z-score, we look at the z-score table and find the p-value associated with this z-score. This p-value is the probability that the value of the measure is smaller than X, that is, the percentile of X. Subtracting 1 by the pvalue, we get the probability that the value of the measure is greater than X.

Central Limit Theorem

The Central Limit Theorem estabilishes that, for a normally distributed random variable X, with mean [tex]\mu[/tex] and standard deviation [tex]\sigma[/tex], the sampling distribution of the sample means with size n can be approximated to a normal distribution with mean [tex]\mu[/tex] and standard deviation [tex]s = \frac{\sigma}{\sqrt{n}}[/tex].

For a skewed variable, the Central Limit Theorem can also be applied, as long as n is at least 30.

For the sum of n variables, the mean is [tex]\mu*n[/tex] and the standard deviation is [tex]s = \sigma\sqrt{n}[/tex]

In this question:

[tex]n = 100, \mu = 100*5 = 500, s = 4\sqrt{100} = 40[/tex]

Approximate the probability that it will take less than 450 days to train all the contestants.

This is the pvalue of Z when X = 450.

[tex]Z = \frac{X - \mu}{\sigma}[/tex]

By the Central Limit Theorem

[tex]Z = \frac{X - \mu}{s}[/tex]

[tex]Z = \frac{450 - 500}{40}[/tex]

[tex]Z = -1.2[/tex]

[tex]Z = -1.2[/tex] has a pvalue of 0.1151

11.51% probability that it will take less than 450 days to train all the contestants.

A researcher collected data on the hours of TV watched per day from a sample of five people of different ages. Here are the results:i Age TV Hrs 1 43 1 2 30 6 3 22 4 4 20 3 5 5 6 1. Calculate the least squares estimated regression equation using simple linear regression. 2. What is the independent variable in this study?a) {y}b) agec) tv hoursd) Ie) 53) Create an ANOVA table. Using α=.05.

Answers

Answer:

1. The least squares regression is y = -0.1015·x + 6.51

2. The independent variable is b) age

Please see attached table

Step-by-step explanation:

The least squares regression formula is given as follows;

[tex]\dfrac{\sum_{i = 1}^{n}(x_{i} - \bar{x})\times \left (y_{i} - \bar{y} \right ) }{\sum_{i = 1}^{n}(x_{i} - \bar{x})^{2}}[/tex]

We have;

[tex]\bar x[/tex] = 24

[tex]\bar y[/tex] = 4

[tex]\Sigma (x_i - \bar x) (y_i - \bar y)[/tex] = -79

[tex]\Sigma (x_i - \bar x)^2[/tex] = 778

[tex]\therefore \hat \beta =\dfrac{\sum_{i = 1}^{n}(x_{i} - \bar{x})\times \left (y_{i} - \bar{y} \right ) }{\sum_{i = 1}^{n}(x_{i} - \bar{x})^{2}} = \frac{-79}{778} = -0.1015[/tex]

The least squares regression is y = -0.1015·x + α

∴ α = y  -0.1015·x = 6 - (-0.1015 × 5) = 6.51

The least squares regression is thus;

y = -0.1015·x + 6.51

2. The independent variable is the age b)

3. Steps to create an ANOVA table with α = 0.05

The overall mean = (43  + 30  + 22  + 20  + 5  + 1  + 6  + 4  + 3  + 6 )/10 = 14

There are 2 different treatment = [tex]df_{treat} = 2 - 1 = 1[/tex]

There are 10 different treatment measurement = [tex]df_{tot} = 10 - 1 = 9[/tex]

[tex]df_{res} = 9 - 1 = 8[/tex]

[tex]df_{treat} + df_{res} = df_{tot}[/tex]

The estimated effects are;

[tex]\hat A_1 = 24 - 14 = 10[/tex]

[tex]\hat A_2 = 4 - 14 = -10[/tex]

[tex]SS_{treat} = 10^2 \times 5 + (-10)^2 \times 5 =1000[/tex]

[tex]\sum_{i}\SS_{row}_i = \sum_{i}\sum_{j} (y_{ij} - \bar y)= [(1 - 4)^2 + (6 - 4)^2 + (4 - 4)^2 + (3 - 4)^2 + (6 - 4)^2] = 18[/tex]

[tex]\sum_{i} S S_{row}_i = \sum_{i}\sum_{j} (y_{ij} - \bar y) ^2= [(43 - 24)^2 + (30 - 24)^2 + (22 - 24)^2 + (20 - 24)^2 + (5 - 24)^2] = 778[/tex]

[tex]S S_{res} = \sum_{i} S S_{row}_i = 778 + 18 = 796[/tex]

[tex]SS_{tot}[/tex] = (43  - 14)² + (30  - 14)² + (22  - 14)² + (20  - 14)² + (5  - 14)² + (1 - 14)1² + (6  - 4 )² + (3  - 14)² + (6  - 14)² = 1796

[tex]MS_{treat} = \dfrac{SS_{treat} }{df_{treat} } = \dfrac{1000}{1} = 1000[/tex]

[tex]MS_{res} = \dfrac{SS_{res} }{df_{res} } = \dfrac{796}{8} = 99.5[/tex]

F- value is given by the relation;

[tex]F = \dfrac{MS_{treat} }{MS_{res} } = \frac{1000}{99.5} = 10.05[/tex]

We then look for the critical values at degrees of freedom 1 and 8 at α = 0.05 on the F-distribution tables 5.3177

Hence; [tex]F = 10.05 > F_{1,8}^{Krit}(5\%) = 5.3177[/tex], we reject the null hypothesis.

The measure of angle 1 is (3x + 10) and the measure of
angle 4 is (4x - 15)
What is the measure of angle 7?
2 4
6 8
5 7
13
b

Answers

Answer:

95

Step-by-step explanation:

just did it on ed

Question one answer: S²=3V/H


What is the opposite operation of squaring? Using this opposite operation, rewrite the equation from question 1 so s is by itself on one side of the equation.


Step by step answer

Answers

Answer:

[tex]\ S=\sqrt{\dfrac{3V}{H}}}[/tex]

Step-by-step explanation:

The opposite operation of squaring is taking the square root.

[tex]\ S=\sqrt{\dfrac{3V}{H}}}[/tex]

We know that the denominator of a fractional power is the index of the corresponding root:

[tex]\displaystyle x^\frac{1}{n}=\sqrt[n]{x}[/tex]

For n=2, we don't usually write the index in the root symbol:

[tex]x^{\frac{1}{2}}=\sqrt{x}[/tex]

In the case of this problem, ...

[tex](S^2)^{\frac{1}{2}}=\left(\dfrac{3V}{H}\right)^{\frac{1}{2}}\\\\S=\sqrt{\dfrac{3V}{H}}[/tex]

PLEEEASE HELPPPPPPP!! substitution

Answers

Answer:

X=80

Y=9

Z=7

Step-by-step explanation:

i got the same answer as the person before

The angle measures associated with which set of ordered pairs share the same reference angle? (Negative StartFraction StartRoot 3 EndRoot Over 2 EndFraction , negative one-half), (negative one-half, Negative StartFraction StartRoot 3 EndRoot Over 2 EndFraction) (one-half, Negative StartFraction StartRoot 3 EndRoot Over 2 EndFraction), (Negative StartFraction StartRoot 3 EndRoot Over 2 EndFraction, one-half) (Negative one-half, negative StartFraction StartRoot 3 EndRoot Over 2 EndFraction), (One-half, StartFraction StartRoot 3 EndRoot Over 2 EndFraction) (StartFraction StartRoot 3 EndRoot Over 2 EndFraction, one-half), (one-half, StartFraction StartRoot 3 EndRoot Over 2 EndFraction)

Answers

Answer:

[tex](C)\left(-\dfrac{1 }{2},-\dfrac{\sqrt{3} }{2} \right)$ and \left(\dfrac{1 }{2},\dfrac{\sqrt{3} }{2} \right)[/tex]

Step-by-step explanation:

The reference angle is the angle that the given angle makes with the x-axis.

For an ordered pair to share the same reference angle, the x and y coordinates must be the same or a factor of each other.

From the given options:

[tex](A)\left(-\dfrac{\sqrt{3} }{2} ,-\dfrac{1 }{2}\right)$ and \left(-\dfrac{1 }{2},-\dfrac{\sqrt{3} }{2} \right)\\\\(B)\left(\dfrac{1 }{2},-\dfrac{\sqrt{3} }{2} \right)$ and \left(-\dfrac{\sqrt{3} }{2}, \dfrac{1 }{2}\right)\\\\(C)\left(-\dfrac{1 }{2},-\dfrac{\sqrt{3} }{2} \right)$ and \left(\dfrac{1 }{2},\dfrac{\sqrt{3} }{2} \right)\\\\(D)\left(\dfrac{\sqrt{3} }{2},\dfrac{1 }{2} \right)$ and \left(\dfrac{1 }{2},\dfrac{\sqrt{3} }{2} \right)[/tex]

We observe that only the pair in option C has the same x and y coordinate with the second set of points being a negative factor of the first term. Therefore, they have the same reference angle.

Answer:

C

Step-by-step explanation:

What would the amplitude be? How do I find it? Should I add 1.25+6.75 then divide by 2 ?

Answers

Answer:

2.75

Step-by-step explanation:

The amplitude is half of the difference between the highest and lowest y-coordinates.

amplitude = 0.5|6.75 - 1.25| = 0.5(5.5) = 2.75

If f(a) = 3a - a2, which of the following are not true statements?
Select all that apply.
f(4) = -4
f(3) = 0
f(-1) = 2
f(0) = 3
f(-5) = -40

Answers

Answer:

f(-1) = 2

f(0) = 3

Step-by-step explanation:

f(a) = 3a - a²

f(4)= 3*4-4²= -4 ⇒  correctf(3)= 3*3-3²= 0 ⇒ correctf(-1) = 3*(-1)-(-1)²=-3+1= -2 ⇒ incorrect f(0) = 3*0-0²= 0 ⇒ incorrect f(-5) = 3*(-5)-(-5)²= -15-25= -40 ⇒ correct

Answer:

The 3rd and the 4th statement are not the true statements.

Step-by-step explanation:

In the 3rd statement, if you put a=-1 into the equation f(a), it would be

=3x(-1) - (-1)2

=-3 - 1

=-4.

In the 4th statement, if you put a=0 into the equation f(a), it would be

=3x(0) - (0)2

=0 - 0

=0.

For the other statements, they show the correct results.

An investigator predicts that dog owners in the country spend more time walking their dogs than do dog owners in the city. The investigator gets a sample of 21 country owners and 20 city owners. The mean number of hours per week that city owners spend walking their dogs is 10.0. The standard deviation of hours spent walking the dog by city owners is 3.0. The mean number of hours country owners spent walking theirs dogs per week was 15.0. The standard deviation of the number of hours spent walking the dog by owners in the country was 4.0. Do dog owners in the country spend more time walking their dogs than do dog owners in the city?




Using an alpha level of .05 (t= 2.32), what is the conclusion you are entitled to draw as a result of this test?

Answers

Answer:

Yes, there is enough evidence to support the claim that dog owners in the country spend more time walking their dogs than do dog owners in the city (P-value=0.0000263).

Step-by-step explanation:

This is a hypothesis test for the difference between populations means.

The claim is that dog owners in the country (sample 2) spend more time walking their dogs than do dog owners in the city (sample 1).

Then, the null and alternative hypothesis are:

[tex]H_0: \mu_1-\mu_2=0\\\\H_a:\mu_1-\mu_2< 0[/tex]

The significance level is 0.05.

The sample 1, of size n1=20 has a mean of 10 and a standard deviation of 3.

The sample 2, of size n2=21 has a mean of 15 and a standard deviation of 4.

The difference between sample means is Md=-5.

[tex]M_d=M_1-M_2=10-15=-5[/tex]

The estimated standard error of the difference between means is computed using the formula:

[tex]s_{M_d}=\sqrt{\dfrac{\sigma_1^2}{n_1}+\dfrac{\sigma_2^2}{n_2}}=\sqrt{\dfrac{3^2}{20}+\dfrac{4^2}{21}}\\\\\\s_{M_d}=\sqrt{0.45+0.762}=\sqrt{1.212}=1.101[/tex]

Then, we can calculate the t-statistic as:

[tex]t=\dfrac{M_d-(\mu_1-\mu_2)}{s_{M_d}}=\dfrac{-5-0}{1.101}=\dfrac{-5}{1.101}=-4.54[/tex]

The degrees of freedom for this test are:

[tex]df=n_1+n_2-1=20+21-2=39[/tex]

This test is a left-tailed test, with 39 degrees of freedom and t=-4.54, so the P-value for this test is calculated as (using a t-table):

[tex]\text{P-value}=P(t<-4.54)=0.0000263[/tex]

As the P-value (0.0000263) is smaller than the significance level (0.05), the effect is significant.

The null hypothesis is rejected.

There is enough evidence to support the claim that dog owners in the country spend more time walking their dogs than do dog owners in the city.

Using the t-distribution, it is found that since the test statistic is t = 4.54 > 2.32, it can be concluded that dog owners in the country spend more time walking their dogs than do dog owners in the city.

At the null hypothesis, we test if dog owners in the country and in the city spend the same amount of time walking their dogs, that is:

[tex]H_0: \mu_{Co} - \mu_{Ci} = 0[/tex]

At the alternative hypothesis, we test if dog owners in the country spend more time, that is:

[tex]H_1: \mu_{Co} - \mu_{Ci} > 0[/tex]

The standard errors are:

[tex]s_{Co} = \frac{4}{\sqrt{21}} = 0.8729[/tex]

[tex]s_{Ci} = \frac{3}{\sqrt{20}} = 0.6708[/tex]

The distribution of the differences has:

[tex]\overline{x} = \mu_{Co} - \mu_{Ci} = 15 - 10 = 5[/tex]

[tex]s = \sqrt{s_{Co}^2 + s_{Ci}^2} = \sqrt{0.8729^2 + 0.6708^2} = 1.1009[/tex]

We have the standard deviation for the samples, hence, the t-distribution is used. The test statistic is given by:

[tex]t = \frac{\overline{x} - \mu}{s}[/tex]

In which [tex]\mu[/tex] is the value tested at the null hypothesis, for this problem [tex]\mu = 0[/tex], hence:

[tex]t = \frac{\overline{x} - \mu}{s}[/tex]

[tex]t = \frac{5 - 0}{1.1009}[/tex]

[tex]t = 4.54[/tex]

Since the test statistic is t = 4.54 > 2.32, it can be concluded that dog owners in the country spend more time walking their dogs than do dog owners in the city.

A similar problem is given at https://brainly.com/question/17192140

Find the average of each of the following.
(a) $357, $452, $589, $602, $775

I need help anybody knows how to do? ​

Answers

Answer:

555

Step-by-step explanation:

(357+452+589+602+775)/5

Step-by-step explanation:

I HAVE DONE AT HERE YOU HAVE ASK WHE\E *O PUT 120°

One cylinder has a volume that is 8 cm' less than a of the volume of a second cylinder. If the first cylinder's volume is 216
cm? what is the correct equation and value of x, the volume of the second cylinder?
x+8-216; x = 182 cm
-8-2167 - 196 cm
coll
+8-216-238cm
col
-8-216
-256 cm
Nacks and otum
Save and Exit
Save and Exit​

Answers

Corrected Question

One cylinder has a volume that is 8cm less than 7/8 of the volume of a second cylinder. If the first cylinder’s volume is 216cm³, what is the correct equation and value of x, the volume of the second cylinder?

Answer:

The equation is: [tex]216=\frac{7}{8}x-8[/tex]

Volume of the second cylinder= [tex]256cm^3[/tex]

Step-by-step explanation:

Volume of the first cylinder=[tex]216cm^3[/tex]

Let the volume of the second cylinder=x

7/8 of the volume of a second cylinder=[tex]\frac{7}{8}x[/tex]

8 less than 7/8 of the volume of a second cylinder=[tex]\frac{7}{8}x-8[/tex]

Therefore, the equation is:

[tex]216=\frac{7}{8}x-8[/tex]

Next, we solve for x

[tex]216=\frac{7}{8}x-8\\216+8=\frac{7}{8}x\\224=\frac{7}{8}x\\$Cross multiply\\7x=224*8\\Divide both sides by 7\\x=256cm^3[/tex]

The volume of the second cylinder is [tex]256cm^3[/tex]

Answer:

The answer is D

Step-by-step explanation:

what is a proportional relationship?

Answers

Proportional relationships are relationships between two variables where their ratios are equivalent. Another way to think about them is that, in a proportional relationship, one variable is always a constant value times the other.

In choosing what music to play at a charity fund raising event, Cory needs to have an equal number of piano sonatas from J. S. Bach, Haydn, and Wagner. If he is setting up a schedule of the 9 piano sonatas to be played, and he has 5 J. S. Bach, 52 Haydn, and 5 Wagner piano sonatas from which to choose, how many different schedules are possible

Answers

Answer:

2,210,000 different schedules

Step-by-step explanation:

Cory needs to have an equal number of piano sonatas from J. S. Bach, Haydn, and Wagner.

Since he is setting up a schedule of 9 piano sonatas to be played, he needs:

3 out of 5 J. S. Bach piano sonatas3 out of 52 Haydn piano sonatas3 out of 5 Wagner piano sonatas

We then calculate how many different schedules are possible using combination.

Number of possible Schedules

[tex]=$ ^5C_3$ \times ^{52}C_3$ \times ^5C_3\\=10 \times 22100 \times 10\\$=2210000 ways[/tex]

There are 2,210,000 different possible schedules.

In a research article, you find that r is reported to be 4.8. How would you interpret this finding?

a. The relationship is reported incorrectly
b. The relationship is strong
c. The relationship is moderate
d. The relationship is weak

Answers

Based on the information given, the correct option is A. The relationship is reported incorrectly.

It should be noted that in a research, the normal value or r can range between -1 to 1. This shows the linear relationship between the variables.

In this case, since r is reported to be 4.8, the relationship is reported incorrectly.

Learn more about regression on:

https://brainly.com/question/25987747

You would interpret this finding of r to be reported as 4.8 as (a) the relationship is reported incorrectly.

The variable r in regression represents correlation.

And in regression, correlation can only take values between -1 and 1 (inclusive)

Given that:

r = 4.8

4.8 is outside the range -1 to 1.

This means that, the value is either calculated incorrectly or reported incorrectly.

Hence, the true statement is (a)

Read more about regression and correlation at:

https://brainly.com/question/14585820

The owner of Limp Pines Resort wanted to know the average age of its clients. A random sample of 25 tourists is taken. It shows a mean age of 46 years with a standard deviation of 5 years. The width of a 95 percent CI for the true mean client age is approximately: _________

Answers

Given Information:

Sample size = n = 25

Mean age of client = 46 years

Standard deviation of age of client = 5 years

Confidence level = 95%

Required Information:

Width of the confidence interval = ?

Answer:

[tex]$ \text {width of CI } = \pm 2.064 $\\\\[/tex]

Step-by-step explanation:

The width for the true mean client age is given by

[tex]$ \text {width of CI } =\pm t_{\alpha/2}(\frac{s}{\sqrt{n} } ) $[/tex]

Where n is the sample size, s is the standard deviation of age of client, and [tex]t_{\alpha/2}[/tex] is the t-score corresponding to 95% confidence level.

The t-score corresponding to 95% confidence level is

Significance level = 1 - 0.95 = 0.05/2 = 0.025

Degree of freedom = n - 1 = 25 - 1 = 24

From the t-table at α = 0.025 and DF = 24

t-score = 2.064

[tex]$ \text {width of CI } = \pm (2.064)(\frac{5}{\sqrt{25} } ) $\\\\[/tex]

[tex]$ \text {width of CI } =\pm (2.064)(\frac{5}{5 } ) $\\\\[/tex]

[tex]$ \text {width of CI } = \pm (2.064)(1) $\\\\[/tex]

[tex]$ \text {width of CI } = \pm 2.064 $\\\\[/tex]

Therefore, width of the 95% confidence Interval for the true mean client age is approximately ±2.064.

Bonus:

The corresponding 95% confidence interval is given by

[tex]$ \text {Confidence Interval } = \bar{x} \pm t_{\alpha/2}(\frac{s}{\sqrt{n} } ) $[/tex]

Where x_bar is the mean client age

[tex]$ \text {Confidence Interval } = 46 \pm 2.064 $[/tex]

[tex]$ \text {Confidence Interval } = (43.936, 48.064) $[/tex]

[tex]$ \text {Confidence Interval } = (44, 48) $[/tex]

Which means that we are 95% sure that the true mean client age is within the range of (44, 48) years.

Give the answer in a + bi form. (−7 + 6i) + (6 − 8i)

Answers

Answer:

-1 -2i

Step-by-step explanation:

(-7+6) + (6i-8i) = -1 -2i

Answer:

(-1 - 2i)

Step-by-step explanation:

You simply add them.

-7 + 6 = -1

6i + (-8i) = 6i - 8i = -2i

Other Questions
Definitions of religions are ___ and are based on ___ In photosynthesis water molecules are split into ____ and _____ atoms1. Hydrogen and oxygen2. Sulfur and ithium3. Carbon and nitrogen4. glucose and fructose ON MONDAY SARAH HAD HOMEWORK IN 7/10 OF HER CLASSES, TUESDAY 3/5, WEDNESDAY 9/11, AND THURSDAY 1/2.WHICH DAY DID SHE HAVE THE MOST HOMEWORK ? Grant has a sample of 36 rats in his study. If he conducts a single-sample t test, what is the value of the degrees of freedom?3763635 There should be 2 paragraphs in this text. Select where the second paragraph begins.My favourite restaurant is a local one that is just across the road from us. Its nothing fancy, but the food is nice and the service is good. I always order the meal of the day. If I had to choose the worst restaurant, Id probably say that it was the Italian nearby. The food is awful and overpriced Please help I dont get how to do it. What is the sequence of energy transformations that occur in a nuclear reactor? nuclear energy Right arrow. Mechanical energy Right arrow. Thermal energy Right arrow. Electrical energy nuclear energy Right arrow. Thermal energy Right arrow. Mechanical energy Right arrow. Electrical energy thermal energy Right arrow. Nuclear energy Right arrow. Mechanical energy Right arrow. Electrical energy electrical energy Right arrow. Thermal energy Right arrow. Mechanical energy Right arrow. Nuclear energy Each statement describes a transformation of the graph of f(x) = x. Which statement correctly describes the graph of g(x) if g(x) = f(6x)? Sodium metal and water react to form hydrogen and sodium hydroxide. If 11.96 g of sodium react with water to form 0.52 g of hydrogen and 20.80 g of sodium hydroxide, what mass of water was involved in the reaction I need help with this ASAP, my assignment is due in 15 minutes. I dont speak Spanish btw. Here are the instructions: For each question you will be iven a set o scrambled words. Unscramble them to orm a Spanish sentence in the present tense and type it in the box under the scrambled words. When you have finished the Spanish sentence, translate it into Enlish below the Spanish sentence. Which linear equation represents a line with a slope of 9/4 and a y-intercept of 0? plz help giving out brainliest Solve the inequality 8(x + 1) > 7(x + 2) Read the following sentence.One time, Heather offered to make jackets for Ana and I.If the pronoun in the sentence above is correct as is, write "correct" and explain why the pronoun choice is correct. If the sentence contains a pronoun error, rewrite the sentence to correct the error, then explain the reason the pronoun was an error. Which line's meter is iambic?O A. Cigarettes to be sure are unhealthy.O B. Create a team that stands as one.O C. Martin isn't ever going fishing now.O D. Music is harmony, rhythm, and melody.SUBT que es el mensaje de geso y pellejo? Write sin 49 in terms of cosine. Authors choose their words for particular effects. Their word choice is called (answers) denotation diction mood tone Explain in detail, how you solved the following problem: The first two terms of a sequence are 10 and 20. If each term after the second term is the average of all of the preceding terms, what is the 2020th term? After Mia gave 1/4 of her milk to Dante, Dante had twice the amount of milk as Mia. What was the ratio of the amount of milk Mia had to the amount of milk Dante had at first? PART A: Which statement identifies the central idea of the text?A.In ancient Egypt, it was very unlikely for nobles to be granted eternal life by thegods.B.In order to be accepted into the afterlife in ancient Egypt, an individual wasjudged by the wealth they possessed at the end of their lives.C. Mummification was a process in which ancient Egyptian citizens showed respectfor their nobles.D. In ancient Egypt, a noble's journey to the afterlife includes mummification andspiritual judgement by the God Osiris.